Densité...

Bonsoir communauté mathématiques, j'ai à vous proposer, un exercice sur lequel je bloque et je ne vois pas trop comment m'y prendre... J'espère pouvoir récolter quelques indications qui pourront mener mon étude de ce problème à bien. Merci de votre aide d'avance. Voici l'énoncé:

"Est-ce que

$ \frac{2^n}{3^k} : n, k \in \N $

est dense dans $[0,\infty[$ ?"

Merci beaucoup pour votre aide précieuse.

CMC

Réponses

  • La fonction $ln$ est un homéomorphisme de $]0;+\infty[$ sur $\R$.

    Ton ensemble est donc dense dans $]0;+\infty[$ si et seulement si $\{ n ln(2) - k ln(3), \, (n,k)\in \N^2 \}$ est dense dans $\R$, ce qui est moins difficile à montrer (en sachant qu'un sous-groupe de $\R$ est soit monogène soit dense).
  • Nous n'avons pas vu en cours ce qu'est un groupe monogène... En vérité cet exercice est tiré d'un TD d'Analyse à Variables Réelles. Et ton idée semble sympathique mais je ne vois toujours pas comment conclure quant à la densité de mon ensemble.
  • Un groupe monogène c'est un groupe engendré par un seul élément ; c'est l'ensemble des puissances de cet élément. Dans le cas des sous-groupes additifs de $\R$, ce sont les sous-groupes discrets de la forme $a \Z$ avec $a \in \R$. Et le résultat de Guégo, que tu ne sembles pas connaître, est que si un sous-groupe de $(\R,+)$ n'est pas de cette forme-là alors il est dense dans $\R$.


    Bon, j'avais une autre idée pour l'exo mais en fait elle ne marche pas, je m'y remets.
  • Bon, alors on va faire ça par étapes :
    Quitte à tout diviser par $ln(2)$, ton problème revient à montrer que $\{ n - k\alpha, \, (n,k)\in \N^2 \}$ est dense dans $\R$, avec $\alpha = \frac{ln(3)}{ln(2)}$.

    1) Montrer que $\alpha$ est irrationnel.
    2) Montrer que pour tout $\epsilon > 0$, on peut trouver $n$ et $k$ tels que
    $|n - k \alpha|< \epsilon$.
    3) Montrer que pour tout $\epsilon > 0$, on peut trouver $n$ et $k$ tels que $0 < n - k \alpha < \epsilon$ et qu'on peut aussi trouver $n$ et $k$ tels que $- \epsilon < n - k \alpha < 0$.
    4) Montrer la densité dans $\R$ de l'ensemble des $n-k\alpha$.

    Remarque : aucune des 4 étapes n'est évidente. L'exercice posé tel quel est rude...
  • Ah comme tu me le dictes là ça m'a l'air déjà plus faisable. Mais posé pour un niveau L2 c'est peut être un peu haut comme niveau non?
  • Oui, c'est ce que je disais : l'exercice d'origine posé tel quel est vraiment difficile. Même détaillé avec mes 4 questions, ça n'est pas évident.
  • rien que le 1) de Guego demande de l'abnégation.
  • Et justement pour l'irrationalité de $\alpha$ je fais comment?

    CMC
  • Toujours pas d'idée?
  • Pour le 1 : Si $\alpha = \dfrac{p}{q}$, alors $2^q = 3^p$. Donc par unicité de la décomposition en facteur premiers...
  • Pourriez vous me rédiger une solution comme cela doit être rédigé s'il vous plait? Notre prof nous a dit qu'on pouvait se faire aider, car ce n'est pas de notre niveau, et que si l'on réussissait (et rendait avant les vacances de Noël) il nous rajouterait certainement des points à l'examen que l'on a fait dernièrement, et ça m'arrangerait un peu je dois avouer. Il nous a dit qu'on pouvait s'aider comme on le souhaitait. J'ai bien essayé de rédiger quelque chose mais je bloque toujours, je compte sur votre aide, je vous en serais reconnaissant.
    En vous remerciant d'avance.
    Charles.
  • Salut ;
    ona aussi le resultat suivant :
    $\D$ est dense dans $\R$ ssi $\forall y \in \R $, $\exists x_n \in \D$ tq $lim x_n = y $, $n \rightarrow \infty$
    $(x_n)$ suite d'elements de $\D$
  • Charles57, as tu montré que pour tout $ \epsilon > 0$, on peut trouver $ n$ et $ k$ tels que
    $ \vert n - k \alpha\vert< \epsilon$.

    après on t'aide.
  • J'aimerais bien réussir à le montrer justement.
  • Tu disposes du résultat sur les sous-groupes de $\R$ ou non ?
  • Non justement le prof n'a pas voulu s'attarder là dessus.
  • S'il vous plait, j'aimerais bien y voir plus clair.
  • Je te propose

    $a^+=Inf\{n-k\alpha,(n,k)\in\N^2 \text{tel que } n-k\alpha>0\}$
    $a^-=Inf\{n-k\alpha,(n,k)\in\Z-^2 \text{tel que } n-k\alpha>0\}$
    On suppose $a^+\leq a^-$

    premier cas, tu supposes que $a^+$ est atteinte par $a^+=n_0-k_0\alpha>0$ avec $(n_0,k_0)\in\N^2$
    soit $(n,k)\in\N^2 \text{tel que } n-k\alpha>0$,
    alors par division euclidienne tu as $n-k\alpha=qa^+ +r$ avec $0\leq r0, à partir d'un certain rang, tu auras $0
  • pour la division euclidienne il faut bien dire que $a^+$ non nul, sinon on ne peut pas diviser par 0.
  • Ca me semble somme toutes compliqué ! Avec mon niveau début L2 je ne pourrais pas faire grand chose de ce que tu me demandes de compléter. De plus j'ai un peu de mal à comprendre la démarche que tu me proposes.
  • Pour motiver un peu la démarche : tu veux montrer que tu peux trouver des éléments de la forme $n-k \alpha$ arbitrairement proches de $0$, ce qui revient à montrer qu'un certain $\inf$ est égal à $0$.
  • j'ai fait ça pour montrer
    3) Montrer que pour tout $ \epsilon > 0$, on peut trouver $ n$ et $ k$ tels que $ 0 < n - k \alpha < \epsilon$ et qu'on peut aussi trouver $ n$ et $ k$ tels que $ - \epsilon < n - k \alpha < 0$.

    pour répondre à cette question je te propose de montrer
    $ Inf\{n-k\alpha,(n,k)\in\mathbb{N}^2$ tel que $ n-k\alpha>0\}=0$
    $ Inf\{n-k\alpha,(n,k)\in\mathbb{Z}-^2$ tel que $ n-k\alpha>0\}=0$

    (j'ai commencé par supposer $a^+\leq a^-$ et on arrive à montrer en effet que $a^+=a^-=0$. si on suppose $a^-\leq a^+$, on fait pareil.)

    c'est mon idée, mais si quelqu'un a une meilleure idée, ça pourait plus te motiver.
  • Ok merci je vais essayer de voir ce que je peux faire.
  • J'ai peut-être une idée un brin naïve du physicien que je suis: 2 et 3 étant deux nombres premiers distincts, il y a bijection entre E={2^k/3^n, (k,n) dans N×N} et l'ensemble des fractions réduites p/q. Ce dernier ensemble étant dense dans R+, E l'est aussi. Non ?
  • malheureusement l'image d'un ensemble dense par une bijection peut être n'importe quoi, aussi bien dense que non dense.
    Q est dense dans R et son cardinal est dénombrable
    inversemment des intervalles ont le cardinal de R et ne sont pas dense dans R
  • La méthode avec les inf est-elle la seule possibilité de régler ce problème car elle me semble très compliquée, n'y aurait-il pas une méthode plus courte?
  • Non, et j'ai donné un truc très semblable en colle la semaine dernière.

    On peut commencer par montrer un résultat intermédiaire :

    Soit $\alpha$ irrationnel. Pour tout $n\in\N^*$, il existe $p_n\in\Z$ et $q_n\in\N^*$ tels que $$\left\lvert\alpha-\frac{p_n}{q_n}\right\rvert
  • Après appui sur je ne sais quelle touche mon texte a disparu...Grrr !!!
    <BR>
    <BR>Je reprends:
    <BR>
    <BR>1) réglé
    <BR>
    <BR>2) n=k=0 solution.
    <BR>
    <BR>3) <SPAN CLASS="MATH"><IMG WIDTH="108" HEIGHT="30" ALIGN="MIDDLE" BORDER="0" SRC="http://www.les-mathematiques.net/phorum/2006/12/12/104137/cv/img1.png&quot; ALT="$ 0<n-k\alpha<\epsilon$"></SPAN>
    <BR>Si on pose <SPAN CLASS="MATH"><IMG WIDTH="91" HEIGHT="32" ALIGN="MIDDLE" BORDER="0" SRC="http://www.les-mathematiques.net/phorum/2006/12/12/104137/cv/img2.png&quot; ALT="$ \alpha=n/k-\epsilon'$"></SPAN>, on obtient <SPAN CLASS="MATH"><IMG WIDTH="42" HEIGHT="32" ALIGN="MIDDLE" BORDER="0" SRC="http://www.les-mathematiques.net/phorum/2006/12/12/104137/cv/img3.png&quot; ALT="$ \epsilon'<\epsilon$"></SPAN> ce qui doit être possible, étant donné qu'on peut prendre un rationnel arbitrairement proche de tout réel, non ?<BR><BR><BR>
  • Charles57, si tu veux utiliser la méthode d'Eric, je pense que la difficulté est la même, mais elle est bien celle là,
    Ce que je t'avais proposé, de toute façon tu ne pourras pas y échapper car c'est comme ça qu'on trouve les sous groupes de R
    Ce que propose Eric, tu le verras sans doute aussi un peu plus tard.
    donc quelle méthode veux tu utiliser ? pour qu'on sache laquelle il faut t'expliquer.
  • La fonction $ ln$ est un homéomorphisme de $ ]0;+\infty[$ sur $ \mathbb{R}$.

    Comment le montre-t-on??

    Sinon on a montré que l'ensemble $ \{ n - k\alpha, \, (n,k)\in \mathbb{N}^2 \}$ est dense dans $\R$ si et seulement si $\alpha$ est un irrationnel, donc en fait ici j'ai juste à montrer que mon apha est irrationnel et ça sera bon?

    CMC
  • Homéomorphisme ou homomorphisme d'ailleurs (Guego parle de groupe) ?
    Sinon ln de R*+ vers R est bijective et continue et sa réciproque exp l'est également, ceci doit suffire, si je ne m'abuse, à montrer que cette application est un homéomorphisme.
  • Ben justement je ne sais pas lequel des deux, faudrait que Guego m'éclaire là dessus ou quiconque qui s'y connaitrait un tant soit peu en groupes et ensembles. Il ne me manque que ça pour parfaire l'exercice demandé pour lequel j'ai passé des heures dessus ^^ J'en vois enfin la fin !
  • Mais sinon c'est bien un homomorphisme de groupes:
    ln de (R*+,×) vers (R,+) vérifie bien f(xy)=f(x)f(y) puisque ln(a×b)=ln(a)+ln(b).
    J'avais appris cette histoire d'homomorphisme en première année de pharma. Souvenirs, souvenirs...
  • si $ \{ n - k ln(3)/ln(2), \, (n,k)\in \mathbb{N}^2 \}$ est dense dans R alors,
    pour tout x>0, il existe deux suites $(n_p)_p$ et $(k_p)_p$ telles que
    $n_p-ln(3)/ln(2) k_p \longrightarrow ln(x)/ln(2)$
    donc $ln(2)n_p-ln(3) k_p \longrightarrow ln(x)$
    et alors comme exponentielle est continue en $ln(x)$, $exp(ln(2)n_p-ln(3)k_p)\longrightarrow exp(ln(x))$
    ou encore $\dfrac{2^{n_p}}{3^{k_p}}\longrightarrow x$
  • on n'a pas de chance, on a pris $\alpha=ln(3)/ln(2)>1$, on aurait pu prendre $ln(2)/ln(3)>1$
    pas grave, je vais prendre $\beta=\alpha-1k_2$ tels que $|\bar{k_1\beta}-\bar{k_2\beta}| \leq 1/p$

    et qu'il existe n et k dans $\N$ tels que $0\leq n-k \beta \leq 1/p$
    et enfin reviens à $\alpha = \beta+1$

    puis montre qu'il existe n et k dans $\N$ tels que $ -1/p \leq n-k\alpha \leq 0$
  • quand j'ai dis il existe n et k dans $ \mathbb{N}$ tels que $ 0\leq n-k \beta \leq 1/p$, il y a une erreur, c'est n et k dans $ \mathbb{N}^*$
  • De même que c'est <SPAN CLASS="MATH"><IMG WIDTH="91" HEIGHT="32" ALIGN="MIDDLE" BORDER="0" SRC="http://www.les-mathematiques.net/phorum/2006/12/15/104470/cv/img1.png&quot; ALT="$ \ln 2/\ln 3<1$"></SPAN>...;-)
  • Bonjour a tous,

    Voici une preuve complete(espece de synthese) qui suit a peu pres le schema de Guego:

    1) Demonstration que$\alpha=\frac{ln(3)}{ln(2)}$est irrationnel

    $2^{\alpha}=3$

    Si $\alpha$ est irrationnel alors $2^{p/q}=3$ donc $2^p=3^q$
    Or la decomposition en facteurs premiers uniques montre que ce n'est pas possible, donc on aboutit a une contradiction.

    2) Pour tout $\epsilon>0$, il existe n, k tels que $n-k\alpha
  • Mon message precedent n'est pas apparu en fin de liste et je ne tiens pas a ce qu'il passe inapercu(ca a ete penible de le taper), donc ce message a pour but de vous rappeler qu'il est la.
  • Encore moi!
    J'oubliais
    Oui, dans ma preuve, le fait que $\alpha$ est irrationnel montre que $n-k\alpha=\epsilon$ est non nul et cela sert a montrer que il existe bien $m$
    pour tout $x$ tel que me
  • Et notre ami Charles devrait récupérer des points... :-)
  • pour anonyme
    tu as recopié ce que j'avais marqué plus haut mais avec moins de détails
    3) $ n-k\alpha$ est dense dans $ \mathbb{R}$

    $ n-k\alpha=\epsilon$ est aussi petit que l'on veut, donc si x est un nombre,
    on a m tel que me
Connectez-vous ou Inscrivez-vous pour répondre.